You are on page 1of 9

CFP Mock Test Retirement Planning

Q.1 An establishment with 100 employees, notified by Central Government and also that which is a
factory engaged in a specified industry, would be covered under Employees Provident fund and
Miscellaneous Provision Act, 1952 if the number of employees, getting salary of Rs.6500 or less, is/are
____________ _. (1)
A) 20 or more
B) It will be covered even if the income of all employees is above Rs. 6500
C) 1 or more
D) More than 20

Q. 2 Which of the following is a defined contribution plan? (1)


A) Leave salary
B) Gratuity
C) Unrecognized Provident Fund
D) Voluntary Retirement Scheme

Q. 3 As an employee Rajesh has come to you with his questions on superannuation plans/ Annuity Plans
to get more educated with current scenario. Deduction in respect of contribution for annuity plan to
certain pension fund under 80C is allowed to a/an __________? (1)
A) Individual resident in Indian
B) Individual Assessee only
C) Individual or HUF
D) Any Assessee

Q.4 As an employee Mohan has come to you with his question on superannuation plans/ Annuity Plans
to get more educated with current scenario. The term 'Employee benefits' includes contributions in
which of the following categories of schemes? (1)
A) Non occupation disability Insurance
B) Unemployment compensation Insurance
C) Social security Scheme
D) Group Insurance Scheme

Q. 5 To what extent the employer.s contribution to the PF and the Superannuation Schemes is treated
as deductible expense? (1)
A) 12% of the salary of the employee
B) 15% of the salary of the employee
C) 24% of the salary of the employee
D) 27% of the salary of the employee

Q.6 Deferred Annuity means ___________. (1)


A) purchase price is paid in a single installment
B) annuity payments begin at the end of a selected term
C) purchase price is paid in regular installments spread through the period of deferment
D) All of the above.

1 Roots Institute of Financial Markets


1197, NHBC Mahavir Dal Road Panipat 132103, Haryana
Ph No. 99961-55000, 99964-55055, Web: www.rifmindia.com, email: info@rifmindia.com
Q.7 Under EPF scheme, monthly pension after retirement is decided on _________. (1)
A) eligible service and pensionable salary
B) pensionable service and pensionable salary
C) actual service and pensionable salary
D) None of the above.

Q.8 The maximum advance a member can take from his/her Provident Fund, for major illness involving
stay in hospital for at least 1 month, is ______. (1)
A) 12 months wage (basic + DA)
B) 24 months wage (basic + DA)
C) 6 months wage (basic + DA)
D) 36 months wage (basic + DA)

Q.9 Sushil has an accumulated amount of Rs 75 lakh at the time of his retirement. Also, just before
retirement, his household expenses are Rs. 2.40 lakh pa and he wishes to maintain the same standard of
living after retirement. If inflation is 6.5% p.a. and interest on investment is 9% pa, how long would this
money last if he gives Rs. 25 lakh to his son out of the accumulated amount? (2)
A) 31.25 Years
B) 30 Years
C) 23 Years
D) None of the above

Q.10 Shyam opens his PPF account in the year 2001- 02. When can he make his first withdrawal from
this PPF account? (2)
A) After 31-03-2006
B) After 31-03-2008
C) After 31-03-2007
D) After 31-03-2016

Q.11 Manish is working in a Private Ltd company covered under PF Act. His salary is Rs. 8500 pm (basic
Rs. 6500 + DA Rs. 2000). His employer contributes towards employee benefits upto the specified salary
limit. What would be the contribution of the employer towards (A) EPS, (B) EPF for Manish? (2)
A) Rs. 541, Rs. 239
B) Rs. 239, Rs. 541
C) Rs. 780, Rs. 239
D) None of the above

Q.12 Amit works as a seasonal employee in an establishment covered under Gratuity Act. His date of
joining is 01 Apr 1977, and works till 31 Mar 2007. His wages (Basic + DA) at the time of retirement is Rs.
10000 per month. Work out his gratuity entitlement. (2)
A) Rs. 115385
B) Rs. 80769
C) Rs. 173077
D) Rs. 350000

2 Roots Institute of Financial Markets


1197, NHBC Mahavir Dal Road Panipat 132103, Haryana
Ph No. 99961-55000, 99964-55055, Web: www.rifmindia.com, email: info@rifmindia.com
Q.13 Brijesh, age 48, plans to retire at 65 and wants to be debt free at retirement. The balance sheet
mortgage is Rs. 114042 at the end of the 10th year of a 30 year loan. The monthly payment was Rs.
953.89. What was the original balance of the loan if the interest rate was 8%? (Select closest answer) (4)

A) Rs. 140000
B) Rs. 130000
C) Rs. 120000
D) Rs. 125000

Q.14 Gary received an inheritance of Rs. 2 Lakh. He wants to withdraw equal periodic payments at the
beginning of each month for 5 years starting after 5 years. He expects to earn 12% annual interest,
compounded monthly on his investments. How much can he receive each month? (4)
A) Rs. 8082.28
B) Rs. 4448.89
C) Rs. 4404.84
D) Rs. 8002.26

Q.15 Alok, age 25 years, plans to retire at age 60 and his life expectancy is 75 years. His current
expenditure is Rs. 200000 annually. He estimates no reduction of expenses post retirement. How much
will he save per annum to achieve his target, if inflation rate is 6% and expected yield from investment is
10%? Assume he wishes to leave an estate of 10% of his savings at the time of retirement. (4)
A) Rs. 9612
B) Rs. 73878
C) Rs. 66490
D) Rs. 8651

Q. 16 Laxman is an NRI who has been working in the US for the past 5 years. He is aged 40 and has a
family of wife and 2 children, aged 14 and 12 to support. He has been saving Rs. 8 lakh per annum for
the past 5 years and hopes to save the same amount for the next 10 years that he plans to live in the US.
He would like to return to India 10 years from now. The inflation-adjusted monthly income requirement
for Laxman, as estimated by the planner, is Rs. 80000 in the year in which he returns to India. It is
estimated that inflation would remain at an average of 3% for the next 30 years. His life expectancy is
placed at 70 years. However if the estimated spend per month, for his family is Rs. 90000 p.m., and the
rate of inflation is 4%, how long will his savings last? Assumption: His investments will earn a rate of
interest of 6%. Compounding to be done on annuity certain basis, through the problem. All numbers
rounded to the nearest five rupees. (4)
A) The funds will last until Laxman is 81 years old. He will not be able to leave behind any estate for his
children.
B) The funds will not be adequate to fund his estimated life span of 70 years. He will leave no estate
behind for the children.
C) The funds will last until Laxman is 72 years old. If his life span is only 70 years, he will leave behind an
estate of Rs. 10904325.
D) The funds will exhaust before Laxman is 71 years old. If his life span is only 70 years, he will leave
Behind an estate of Rs. 987835.

3 Roots Institute of Financial Markets


1197, NHBC Mahavir Dal Road Panipat 132103, Haryana
Ph No. 99961-55000, 99964-55055, Web: www.rifmindia.com, email: info@rifmindia.com
Q. 17 For defined benefit plans, which of the following changes in actuarial assumption Would increase
plan costs to an employee?
1. Early retirement without a reduction in benefits.
2. Longer life expectancy predicted
3. An increase in inflation expectations regarding labor costs. 1

A 1 & 2 only
B 1 only
C 1, 2 & 3
D 2 & 3 only

Q. 18 Sundar has been an employee of a public sector undertaking for the past 25 years And is retiring
the next year. He is eligible for gratuity as per the provisions of the Payment of Gratuity Act, 1972. He
hopes to invest the proceeds along with the PF proceeds, in order to fund his retirement. Sundar does
not receive his gratuity 45 days after his retirement. What are his rights? 1
A Sundar should receive his gratuity within 30 days after his retirement. The Employer has to pay
penal interest for delay in the payment of gratuity.
B Sundar should receive his gratuity within 60 days after his retirement. He Should wait for his
settlement.
C Sundar should receive his gratuity within 21 days after his retirement. The Employer has to pay penal
interest at 15% for delay in the payment of Gratuity.
D There is no legally determined time frame for payment of gratuity. Sundar Has a legal remedy if
payment is not made "within reasonable time."

Q. 19 What options does an employer have for superannuation arrangement?


Option (A) - Payment by employer
Option (B) - Funding through a trust. 1

A Only (A)
B Only (B)
C Neither (A) nor (B)
D Both (A) & (B)

Q. 20 The maximum amount payable under group gratuity is _________. 2


A Rs. 3 lakh
B Rs. 3.50 lakh
C Rs. 2.5 lakh
D 5% of salary

Q.21 Ajay is a small scale trader in nuts and has been making a modest profit for the Past 12 years. He
has pre-dominantly used the PPF to save for his future. He also has a welfare scheme for his few
employees, for whom he runs a small provident Fund, which is yet to be recognized. He also contributes
to this fund as an Employer and matches the employee's contribution. He has encouraged his Employees
also to open PPF accounts to save tax and set aside funds for the Future. Ajay has called you to take a
look at what he has been doing and advise him on the choice of products for providing retirement
benefits to himself and his Employees. Which of the following describes the status of the actual
retirement Benefits received by Ajay's employees from the unrecognized PF account, on Retirement? 2
4 Roots Institute of Financial Markets
1197, NHBC Mahavir Dal Road Panipat 132103, Haryana
Ph No. 99961-55000, 99964-55055, Web: www.rifmindia.com, email: info@rifmindia.com
A The employee's own contribution will be fully tax exempt. The employer's
contribution and interest are taxable under the head 'salary'.
B The entire proceeds are exempt from tax.
C The entire proceeds are subject to tax.
D The employee and employers. contribution are exempt. Interest earned is taxable.

Q.22 Which of the following statement is correct? 2

A Payment under a deferred Annuity Plan starts on retirement


B Payment under a deferred Annuity Plan starts after a specified term
C Payment under a deferred Annuity Plan starts from the age 55 years
D All the above

Q.23 Ahmed.s current annual expenditure is Rs. 100000/-. He is 30 years old and expects to retire at age
55. His annual expenses are estimated to rise by 6% p.a. and his life expectancy is 75 years. His post
retirement annual expenses are estimated to be 80% of his pre-retirement expenses. What will be his
expenses on the first year of his retirement? 4
A Rs. 3,47,425/-
B Rs. 3,43,350/-
C Rs. 4,29,187/-
D Rs. 4,02,350/-

Q.24 Mr. Nirav an affluent broker wants to retire at 45. He wants to maintain his present living standard.
He spends Rs.3,25,000/- a year. He is expected to live upto 85. Inflation is to be assumed at 4% and
expected returns are 7% p.a. How can he achieve this? He is at present 30 years old. What is the nest
egg required at age 45 and what amount shall he save every year to meet his plan? His present
investments are Rs.10,00,000/-.(Assumption: All computations for interest, spend and savings
compound annually, assuming beginning of the period investment).4
A Nest egg and savings required will be Rs.1, 47, 73,065/- and Rs.7, 40,530/- Respectively.
B Nest egg and savings required will be Rs.1, 41, 82,819/- and Rs.4, 24,865/- Respectively.
C Nest egg and savings required will be Rs.1, 27, 73,065/- and Rs.4, 38,300/- Respectively.
D Nest egg and savings required will be Rs.2, 51, 00,065/- and Rs.5, 41,093/- Respectively.

Q. 25 Ms. Rekha is 45 years old and plans to retire at 50. Her life expectancy is 70 years. Ms. Sushma,
her Financial Planner, estimates that her client will require Rs.45,000/- in the first month after
retirement. Inflation rate is 4% p.a. and the rate of return is 6% p.a. What will be the savings per year
required in order to meet this? 4
A Rs. 15,90,000/- (Approx)
B Rs. 14,80,000/- (Approx)
C Rs. 16,90,000/- (Approx)
D Rs. 12,40,000/- (Approx)

Q.26 An eligibility criterion for withdrawal from a Provident fund for purchase of a house is
______________. (1)
A) membership of the fund for 5 years, minimum balance in members a/c Rs. 1,000, purchase
should be in favour of the member or member and spouse.

5 Roots Institute of Financial Markets


1197, NHBC Mahavir Dal Road Panipat 132103, Haryana
Ph No. 99961-55000, 99964-55055, Web: www.rifmindia.com, email: info@rifmindia.com
B) membership of the fund for 10 years, minimum balance in members a/c Rs. 1,000, purchase
should be in favour of the member or member and spouse.
C) membership of the fund for 5 years, minimum balance in members a/c Rs. 5,000, purchase
should be in favour of the member or member and spouse.
D) membership of the fund for 5 years, minimum balance in members a/c Rs. 1,000, purchase
should be in favour of the member or spouse.

Q.27 Which amongst the following is true about the interest received from Recognised Provident
Fund (RPF)? (1)

A) Not treated as income in the year of credit.


B) Not treated as income if the rate is equal to/less than 8.5%. Rate over and above this is
taxable
C) Fully exempt from tax.
D) None of the above

Q.28 Investment in Senior Citizens Pension Plan qualifies for deduction u/s 80C of IT Act. (1)
A) True
B) False
C) Only upto Rs. 70,000
D) Data insufficient

Q.29 Which one of the following statements is NOT true for a defined-benefit plan.
(i) It favors older participants.
(ii) Contributions are annual & arbitrary.
(iii) It requires an actuary. (1)

A) (i) only
B) (ii) only
C) (iii) only
D) None of the statements

Q.30 Senior Citizen.s Savings Scheme is governed by which Act? (1)


A) Small Savings & Misc. Savings Act, 1902
B) Banking Regulation Act, 1949
C) Government Savings Bank Act, 1873
D) Companies Act, 1956

Q.31 Interest rate on Post Office Savings Bank is payable__________. (1)


A) monthly
B) quarterly
C) half yearly
D) Yearly

Q.32 Mr. Prasad is a small scale trader. He has a welfare scheme for his few employees, for whom he
runs a small provident fund, which is yet to be recognised. He also contributes to this fund as an
employer and matches the employees. contribution. He has encouraged his employees to open

6 Roots Institute of Financial Markets


1197, NHBC Mahavir Dal Road Panipat 132103, Haryana
Ph No. 99961-55000, 99964-55055, Web: www.rifmindia.com, email: info@rifmindia.com
PPF accounts to save tax and set aside funds for the future. Can Mr. Prasad open joint PPF
accounts with his employees? (2)
A) Yes, it can be opened, but the proceeds cannot be made payable to either or survivor.
B) A PPF account cannot be opened in joint names.
C) No, joint accounts can be held only by members of the same family or HUFs.
D) Yes, provided the employee is the first holder.

Q.33 Which of the following statement is correct? (2)


A) Payment under a deferred Annuity Plan starts after a specified term.
B) Payment under a deferred Annuity Plan starts on retirement.
C) Payment under a deferred Annuity Plan starts from the age of 55 years.
D) Payment under a deferred Annuity Plan starts immediately on payment of premium

Q.34 Vinita was recently divorced and has two children. The divorce decree requires that she pay 1/3
of the college tuition cost for her children. The tuition cost is currently Rs. 15,000 per year and
has been increasing at 7% per year. Her son and daughter are 12 and 16 respectively and will
attend college for four years beginning at age 18. How much should she save each month,
beginning today for the next five years to finance education for both the children (in nearest
rupee)? Assume that her after-tax rate of return will be 9% and that general inflation has been
4% p.a. (4)
A) Rs. 750
B) Rs. 745
C) Rs. 2,235
D) Rs. 2,500

Q.35 Nirav, aged 30, wants to retire at 45. He wants to maintain his present living standard. He
spends Rs. 3,25,000 a year. He is expected to live upto 85. Inflation is to be assumed at 4% p.a
and expected returns are 7% p.a. What is the nest egg required by Nirav at his age 45 and what
amount should he save every year end to meet his plan? His present investments are Rs.
10,00,000. (4)
A) Nest egg and savings required will be Rs. 1,41,82,828 and Rs. 4,54,606 respectively.
B) Nest egg and savings required will be Rs. 1,47,73,065 and Rs. 7,40,530 respectively.
C) Nest egg and savings required will be Rs. 1,27,73,065 and Rs. 4,38,300 respectively.
D) Nest egg and savings required will be Rs. 2,51,00,065 and Rs. 5, 41,093 respectively.

Q.36 Laxman is an NRI who has been working in the US for the past 5 years. He is aged 40. He has
been saving Rs. 8 lakh per annum for the past 5 years and hopes to save the same amount for the
next 10 years that he plans to live in the US. He would like to return to India 10 years from now.
The inflation-adjusted monthly income requirement for Laxman, as estimated by the planner, is
Rs. 80,000 in the year in which he returns to India. It is estimated that inflation would remain at
an average of 3% p.a for the next 30 years. His life expectancy is placed at 70 years. However if
the estimated spend per month, for his family is Rs. 90,000 p.m., and the rate of inflation is 4%
p.a, how long will his savings last?Assumption: His investments will earn a rate of interest of 6% pa.
Compounding to be done on
annuity certain basis throughout the problem. (4)
A) The funds will last until Laxman is 81 years old.
B) The funds will not be adequate to fund his estimated life span of 70 years.

7 Roots Institute of Financial Markets


1197, NHBC Mahavir Dal Road Panipat 132103, Haryana
Ph No. 99961-55000, 99964-55055, Web: www.rifmindia.com, email: info@rifmindia.com
C) The funds will last until Laxman is 72 years old.
D) The funds will exhaust before Laxman is 71 years old.

Q.37 Ajay has working in two companies viz. A Co. and B Co. He retires from A Co. on November
30, 1988 (salary at the time of retirement Rs. 2,600 per month) and receives Rs. 22,000 as
gratuity out of which Rs. 20,000 was exempted. He also retires from B Co. on December 10,
2006 after 28 years and 8 months of service and receives Rs. 2,90,000 as death-cum-retirement
gratuity. His average basic salary drawn from B Co. for the preceding 10 months ending on
November 30, 2006 is Rs. 18,200 per month. Further he receives Rs. 1,000 per month as DA,
80% of which forms a part of his retirement benefits and 6% commission on turnover achieved
by him. Total turnover achieved by him during 10 months ending November 30, 2006 is Rs.
2,00,000. What is the gratuity exempted from tax for Ajay for AY 2007-08? Assume neither of
the companies A and B are covered under payment of Gratuity Act 1972. (4)
A) Rs. 2,80,000
B) Rs. 2,82,800
C) Rs. 3,30,000
D) Rs. 2,62,800

Q. 38 Savita.s monthly contribution to her company Provident Fund exceeds the statutory requirement
by Rs. 5,000. She plans to visit her son in the United States soon after retirement. If she does not claim
her funds soon after Retirement, what are her rights? (1)
A) The fund will accumulate interest for a period of six months, beginning one month after retirement.
No Interest is payable after this period.
B) The fund will continue to accumulate interest and she can claim it any time after her return.
C) The fund will accumulate interest for one year, from the date of retirement.
D) The fund will stop accumulating interest from the date of her retirement.

Q. 39 The process of retirement planning would generally not involve ________. (1)
A) Concentrating on maximizing returns on Present / Past Investments
B) Projecting individual needs and goals into the future and making sound financial plan
C) Making a plan for management & disposition of assets at death
D) Planning for staying physically healthy & making necessary psychological adjustment and to plan for
housing And leisure / work

Q. 40 Alok, age 25 years, plans to retire at age 60 and his life expectancy is 75 years. His current
expenditure is Rs. 2, 00,000 annually. He estimates no reduction of expenses post retirement. How
much will he save per annum to Achieve his target, if inflation rate is 6% and expected yield from
investment is 10%? Assume he wishes to leave an Estate of 10% of his savings at the time of retirement.
(4)
A) Rs. 73,878
B) Rs. 9,612
C) Rs. 66,490
D) Rs. 8,651

8 Roots Institute of Financial Markets


1197, NHBC Mahavir Dal Road Panipat 132103, Haryana
Ph No. 99961-55000, 99964-55055, Web: www.rifmindia.com, email: info@rifmindia.com
9 Roots Institute of Financial Markets
1197, NHBC Mahavir Dal Road Panipat 132103, Haryana
Ph No. 99961-55000, 99964-55055, Web: www.rifmindia.com, email: info@rifmindia.com

You might also like